You guessed the height of the building to be 23 feet, but it was actually 35 feet. What was your percent error? Round to the nearest WHOLE percent
please round to WHOLE

Answers

Answer 1
The percent error is 34.28 %.
Hope this helps! Be safe out there.
Answer 2

Answer:

34%

Step-by-step explanation:

35 - 23 = 12, so the guess is off by 12/35.

To find the percentage, divide 12 by 35, which is approximately 34%.


Related Questions

Jared made a 15 minute phone call yesterday that cost $0.90. how long will jared be on the phone today if the call cost $1.32, assuming the rate of the cost remains constant

Answers

1. figure out cost per min

.90/15 =.06/min

2. figure out mins. since we know cost per min we can divide that by jared’s total cost for that call

1.32/.06= x mins ——->22 mins

You are _____ to commit a Type I error using the 0.05 level of significance than using the 0.01 level of significance. Group of answer choices more likely less likely equally likely twice as likely

Answers

More likely -You are more likely to commit a Type I error

According to statement

we have to find the type 1 error by using the significant levels.

we are commit a MORE LIKELY to find the error by rejecting a true null hypothesis using the 0.05 level of significance than using the 0.01 level of significance.

So, More likely -You are more likely to commit a Type I error

Learn more about HYPOTHESIS here https://brainly.com/question/11555274

#SPJ4

Quick algebra 1 question for 50 points!

Only answer if you know the answer, quick shout-out to Yeony2202, tysm for the help!

Answers

Answer:

Yes.

Step-by-step explanation:

Yes, the given line can be used to make reasonable predictions of the number of cheese pizzas that will be sold, since the scatter points clearly show a trend.

As time goes on, the number of pizzas sold is getting bigger, and the trend is what created the line.

The data was taken over 8 consecutive weeks, so we can assume that it can be used in our prediction.

Since the trend has been pretty consistent over periods of time, we can expect it to continue in the upcoming weeks.

A number is chosen at random from the first 100 positive integers. find the probability that the number is divisible by 5. 1/10 1/5 1/4

Answers

The probability that the number chosen is divisible by 5 is; 1/5.

What is the probability that the number chosen is divisible by 5?

It follows from the task content that the number of possible outcomes in the probability event is; 100. Additionally, the number of required outcomes is 20 as only 20 numbers are divisible by 5.

Ultimately, the required probability is; 20/100 = 1/5.

Read more on probability;

https://brainly.com/question/7965468

$SPJ1

On a coordinate plane, line k l goes through (negative 6, 8) and (6, 0). point m is at (negative 4, negative 2). which point could be on the line that is parallel to line kl and passes through point m?

Answers

The equation of the line parallel to the line KL passing through the points (-6, 8) and (6, 0), and going through the point M (-4, -2) is 2x + 3y = -6. The point satisfying this equation is (-6, 2). Thus, the 2nd option is the right choice.

Slope of the line KL = (8 - 0)/(-6 - 6) = 8/(-12) = -2/3.

Using the formula of the slope of the line passing through the points (x₂, y₂) and (x₁, y₁), as m = (y₂ - y₁)/(x₂ - x₁).

The slope of the line parallel to the line KL will be equal to the slope of KL.

Thus, the slope of the required line is m = -2/3.

The required line also passes through point M (-4, -2).

Thus, the equation of the required line will be,

y - (-2) = (-2/3)(x - (-4)) {Using the equation of the line passing through the point (x₁, y₁) and a slope m, as y - y₁ = m(x - x₁)},

or, y + 2 = (-2/3)(x + 4),

or, 3(y + 2) = -2x - 8,

or, 2x + 3y = -6.

From the given points, only the point (-6, 2) satisfies the equation.

Thus, the equation of the line parallel to the line KL passing through the points (-6, 8) and (6, 0), and going through the point M (-4, -2) is 2x + 3y = -6. The point satisfying this equation is (-6, 2). Thus, the 2nd option is the right choice.

Learn more about parallel lines at

https://brainly.com/question/13763238

#SPJ4

The provided question is incomplete. The complete question is:

"On a coordinate plane, line K L goes through (negative 6, 8) and (6, 0). Point M is at (negative 4, negative 2). Which point could be on the line that is parallel to line KL and passes through point M?

(–10, 0)

(–6, 2)

(0, –6)

(8, –10)"

solve for x

A)8
B)9
C)-9
D)-8

Answers

Answer:

C) -9

Step-by-step explanation:

x + 19 must be half x + 29 because triangles RBC and RST are similar by SAS and RB is half of RS and RC is half of RT.

2(x + 19) = x + 29

2x + 38 = x + 29

x + 38 = 29

x = -9

Are the ratios 2:1 and 14:6 equivalent?
yes
no

Answers

Answer:

Step-by-step explanation:

To check this, we need to know whether

2

1

=

3

2

We have,  

2

1

=

2×3

1×3

=

6

3

and  

3

2

=

3×2

2×2

=

6

4

We find that

6

3

<

6

4

, which means that  

2

1

<

3

2

.

Therefore, The ratio 1:2 is not equivalent to the ratio 2:3

Answer:

No, they aren't equal.

Step-by-step explanation:

Line a is perpendicular to line b. if the slope of line a is 7 what is the slope of line b

Answers

Answer:

1/7

Step-by-step explanation:

When 2 lines are perpendicular the gradient of one is the reciprocal of the other :

If the gradient of A was 2 the gradient of B would be 1/2.

If the gradient of a is 7 the gradient of b would be 1/7 .

Hope this helped and have a good day

PLEASE HELP ASAP!!!!

Answers

The location of the points on Cartesian plane are listed below:

Figure 1: A(x, y) = (0, 4), B(x, y) = (5, 0)

Figure 2: A(x, y) = (8.5, 4), B(x, y) = (2.5, 1.5)

How to write coordinates in a rectangular system

A point on a Cartesian plane can be represented by rectangular system of coordinates with the form:

(x, y)

Where:

x - Distance along the x-axis respect to origin.y - Distance along the y-axis respect to origin.

Now we proceed to represent each point:

Figure 1: A(x, y) = (0, 4), B(x, y) = (5, 0)

Figure 2: A(x, y) = (8.5, 4), B(x, y) = (2.5, 1.5)

To learn more on Cartesian planes: https://brainly.com/question/27927590

#SPJ1

N=
Help me please!!! Asap thanks so much :)

Answers

Answer:

n = 100

Step-by-step explanation:

the area (A) of the sector is calculated as

A = area of circle × fraction of circle

given A = 40π , then

πr² × [tex]\frac{n}{360}[/tex] = 40π

π × 12² × [tex]\frac{n}{360}[/tex] = 40π

144π × [tex]\frac{n}{360}[/tex] = 40π ( divide both sides by π )

144 × [tex]\frac{n}{360}[/tex] = 40 ( multiply both sides by 360 to clear the fraction )

144n = 14400 ( divide both sides by 144 )

n = 100

Joe is thinking of an integer. The opposite of his number is greater than
the number itself. Which of the following must be true of Joe’s number?

Answers

Answer: Joe's number is a negative number.

Since the number's opposite is greater in value, the number's value is in the negatives.

In a right triangle, one acute angle is 22 degrees less than 3 times the other acute angle. What is the measure of the larger acute angle

Answers

The measure of the acute angles are 62° and 28° .

What is the larger acute angle in a right triangle?The Triangle Angle Sum Theorem states that the sum of all interior angles in a triangle is . Additionally, the Right Triangle Acute Angle Theorem states that the two non-right angles in a right triangle are acute; that is to say, the right angle is always the largest angle in a right triangle.

Let x represent the measure of one of the acute angles of the right angle triangle.

Let y represent the measure of the other acute angle of the right angle triangle.

One of the acute angles in a right triangle measures 22 degrees less than 3 times the other acute angle. This means that

x = 3y - 22

The sum of two acute angles in a right triangle is 90 degrees.

We write as

x + y = 90 - - - - - - - (1)

Substituting x = 3y - 22 into equation 1, it becomes

3y - 22 + y = 90

4y = 90 + 22= 112

y = 112/4 = 28

x = 90 - y = 90 - 28

x = 62

Learn more about  acute angle in a right triangle

brainly.com/question/4174850

#SPJ4

A classroom contains an equal number of boys and girls. if 10 girls leave, twice as many boys as girls remain. what was the original number of students present?

Answers

the answer of this question is 38 and i hope it s  correct

Help please!!!!
Will give brainliest

Answers

The amplitude, Period, Horizontal Shift and Vertical Shift of both graphs are as detailed below.

How to interpret features of trigonometric Graphs?

3)a. Amplitude is the distance between the center line and the positive or negative peak of the graph. Thus, the amplitude is 0.5.

b. The period is the time it takes for the graph to repeat or complete one cycle and in this graph, it is 3.

c. Looking at the graph, ideally the coordinate (3, -1.5) should have been on the y-axis which is at (0, -1.5). This means it was shifted by 3 units to the right side.

d. The positive peak should be equal to the negative peak but in this case, it is not so and as such the center should be on the x-axis which means that the graph was shifted down vertically by 2 units.

4) a. Amplitude is the distance between the center line and the positive or negative peak of the graph. Thus, the amplitude is 3 - 1 = 2

b. The period is the time it takes for the graph to repeat or complete one cycle and in this graph, it is 6.

c. Looking at the graph, ideally the coordinate (4, -1) should have been on the y-axis which is at (0, -1). This means it was shifted by 4 units to the right side.

d. The positive peak should be equal to the negative peak but in this case, it is not so and as such the center should be on the x-axis which means that the graph was shifted up vertically by 1 unit.

Read more about trigonometric graph features at; https://brainly.com/question/17075439

#SPJ1

can someone pls help me on this

Answers

Answer:

The answer is X=3/4 or -1/2

Drag the values to the correct locations to write this series using sigma notation and find the sum of the terms. Not all values will be used.

45 + [45 − 2.5] + ⋯ + [45 − 4(2.5)]

Answers

Answer:

[tex]\displaystyle \\\sum_{k=0}^4 45-2.5k=200[/tex]

Step-by-step explanation:

We start off with the term 45 and decrease by 2.5 each term, ending with four 2.5's subtracted from 45.

Therefore, our index starts at k=0 and ends at 4. The general term can be defined then as [tex]\displaystyle \\\sum_{k=0}^4 45-2.5k[/tex] and evaluating yields 200.

make m the subject!

Answers

how to:
a+b-mx=0
a+b=mx
(a+b)/x=m

7. In a regular polygon, adjacent interior and exterior angles are congruent
What is the name of the polygon?
(1) triangle
(2) rectangle
(4) hexagon
lor
(3) square

Answers

Answer:

This would be true for a rectangle and a square.

Step-by-step explanation:

Rectangles and square have 90 angles.  The exterior angles would also equal 90 because 90 + 90 is 180.  An interior angle and the exterior angle are supplements (add together to 180).

Refer to Table 9-8. Suppose that the data in the table above reflect the price levels in the economy. What is the inflation rate in between 2017 and 2018

Answers

The inflation rate between 2017 and 2018 is 2.86%.

What is the inflation rate?

Inflation rate is used to determine the rate at which the general price levels in an economy is increasing with the passage of time.

The inflation rate = (2018 CPI / 2017 CPI) - 1

(180 / 175) - 1 = 0.0286 = 2.86%

To learn more about CPI, please check: https://brainly.com/question/26382640

#SPJ1

Find the mode of the following data: 18, 17, 12, 14, 8, 21, 10, 11, 19, 20, 10, 5, 17, 12, 10, 20  A. 16  B. 14  C. 13  D. 10

Answers

Answer:

D.10

Step-by-step explanation:

Mode is the number that is repeated the most as we can see the number 10 is there 3 times in the data. 13 isn't there, 14 is only there once and 16 is also not there

The mode would be answer D, 10

Oops, looks like I’m stuck. Can someone please explain to me in detail why 1 plus 1 isn’t 3? (Giving brainliest to the lengthiest and best answer)

Answers

By assuming that 1 + 1 = 3, we will reach an absurd.

How to prove that 1 + 1 isn't 3?

To do this, we need to reach an absurd.

Let's assume that we are working with integer numbers, and let's assume that 1 + 1 is equal to 3.

Then we can write:

1 + 1 = 3

Now, if we subtract 1 in both sides, we get:

(1 + 1) - 1 = 3 - 1

1  = 2

Now we can subtract 1 again so we get:

1 - 1 = 2 - 1

0 = 1

This is an absurd, then the statement:

1 + 1 = 3

is false.

So we conclude that:

1 + 1 isn't 3.

If you want to learn more about math statements:

https://brainly.com/question/1885950

#SPJ1

[tex]\frac{2x}{x+1} +\frac{3(x-1)}{x} =5[/tex]

Answers

Answer:

[tex]x=-3[/tex]

Step-by-step explanation:

Given equation:

[tex]\dfrac{2x}{x+1}+\dfrac{3(x+1)}{x}=5[/tex]

Make the denominators the same:

[tex]\implies \dfrac{2x}{x+1} \cdot\dfrac{x}{x}+\dfrac{3(x+1)}{x} \cdot \dfrac{x+1}{x+1}=5[/tex]

[tex]\implies \dfrac{2x^2}{x(x+1)}+\dfrac{3(x+1)^2}{x(x+1)} =5[/tex]

Combine the fractions:

[tex]\implies \dfrac{2x^2+3(x+1)^2}{x(x+1)} =5[/tex]

Multiply both sides by x(x+1):

[tex]\implies 2x^2+3(x+1)^2 =5x(x+1)[/tex]

Expand the brackets:

[tex]\implies 2x^2+3(x^2+2x+1) =5x^2+5x[/tex]

[tex]\implies 2x^2+3x^2+6x+3 =5x^2+5x[/tex]

Combine like terms:

[tex]\implies 5x^2+6x+3=5x^2+5x[/tex]

Subtract 5x² from both sides:

[tex]\implies 6x+3=5x[/tex]

Subtract 5x from both sides:

[tex]\implies x+3=0[/tex]

Subtract 3 from both sides:

[tex]\implies x=-3[/tex]

Learn more about algebraic fractions here:

https://brainly.com/question/27943438

https://brainly.com/question/27979364

Ali and Baba discovered an underground palace and set out to map all the treasure in it. After carefully mapping the interior, they found that the palace had a vault filled with treasure chests. In each chest, they found either 7 gold bars or 11 silver bars. There were 90 bars in total. How many of them were gold bars?

Answers

Answer:

396 gold bars

Step-by-step explanation:

first of all we will find numbers of gold bar 7*99

which is 693 and

we will find number of silver bars 11*99

which is 1089

then 1089-693=

396

Quick algebra 1 question for 25 points!

Only answer if you know the answer, quick shout-out to Dinofish32, tysm for the help!

Answers

Yes

It's inverse variation

See

3 contractors took 8hours

4 contractors took 6hours

We observe

No of contracter increases hence time decreases

So it's inverse variation

Answer:

[tex]\fbox {Yes}[/tex]

Step-by-step explanation:

An inverse variation can be seen when two factors are compared, if one factor is seen to be increasing, then naturally the factor should be decreasing.

Here, as the number of contractors increase, the time taken to complete the work decreases.

∴ Hence, this represents an inverse variation.

The ratio of oil vinegar in a certain salad dressing is 8:5. How much oil must be blended with 7 liters of vinegar for that recipe?

Answers

the answer is 1:4 hope this helps
Answer is 11.2 liters of vinegar
Step by step
8:5 is 8 oil / 5 vinegar
So we know
8/5 = x/7
Cross multiply 5x = 56
Solve for x by dividing both sides by 5
X = 11.2 liters vinegar

Please help meeeeeeeeeeeeeeeeeeeeeeeeeeeeeeeeeeeeeeeeeee

Answers

Answer:

a) x = 1.5 or x = -0.3

b) x = 5 or x = -8

Explanation:

Quadratic formula:

[tex]\sf x = \dfrac{ -b \pm \sqrt{b^2 - 4ac}}{2a} \quad when \:\: ax^2 + bx + c = 0[/tex]

Here given equation: 5x² - 6x  - 2 = 0

Identify variable constants: a = 5, b = -6, c = -2

Putting these values into equation:

[tex]\sf x = \dfrac{ -(-6) \pm \sqrt{(-6)^2 - 4(5)(-2)}}{2(5)}[/tex]

[tex]\sf x = \dfrac{ 6 \pm \sqrt{36 + 40}}{10}[/tex]

[tex]\sf x = \dfrac{ 6 \pm \sqrt{76}}{10}[/tex]

[tex]\sf x = \dfrac{ 3\pm \sqrt{76}}{5}[/tex]

[tex]\sf x = \dfrac{ 3+ \sqrt{76}}{5} \quad or \quad \dfrac{ 3- \sqrt{76}}{5}[/tex]

In one decimal point:

[tex]\sf x = 1.5 \quad or \quad -0.3[/tex]

b) Here use "middle term split" method

⇒ x² + 3x = 40

relocate

⇒ x² + 3x - 40 = 0

The factors of 40 are 8 and 5

⇒ x² + 8x - 5x - 40 = 0

factor common terms

⇒ x(x + 8) - 5(x + 8) = 0

collect into groups

⇒ (x - 5)(x + 8) = 0

set to zero

⇒ x - 5 = 0 or x + 8 = 0

relocate

⇒ x = 5 or x = -8

Step-by-step explanation:

a) The quadratic formula can help find answers to a quadratic equation when it is equal to 0. The formula is [tex]x=\frac{-b\pm\sqrt{b^2-4ac}}{2a}[/tex] for a quadratic equation of the form [tex]ax^2+bx+c=0[/tex].

In this question, a is 5, b is -6, and c is -2. Let's put them in the equation and solve.

[tex]x=\frac{6\pm\sqrt{(-6)^2-4(5)(-2)}}{2(5)}\\x=\frac{6\pm\sqrt{36+40}}{10}\\x=\frac{6\pm\sqrt{76}}{10}\\x=\frac{6\pm2\sqrt{19}}{10}\\x=\frac{3\pm\sqrt{19}}{5}[/tex]

This means that the value of x is both [tex]\frac{3+\sqrt{19}}5[/tex] and [tex]\frac{3-\sqrt{19}}5[/tex], since both values make x equal to 0. Putting both into the calculator, we get that [tex]x=1.5[/tex] or [tex]x=-0.3[/tex].

b) We can easily solve this equation using factoring, which turns a quadratic into two factors, and finds the solutions by setting both to 0. This will only work if the quadratic is equal to 0.

[tex]x^2+3x-40=0[/tex]

Now, we have to find two numbers that add to 3 and multiply to -40. The numbers 8 and -5 work, as 8-5=3 and 8*-5 is -40.

we can now make our factors x+8 and x-5

[tex](x+8)(x-5)=0[/tex]

If two numbers, say A and B, are being multiplied and equal 0, then either A is 0, or b is 0, or both. Similarly, if x+8 and x-5 are being multiplied and equal 0, either x+8 = 0, or x-5=0.

[tex]x+8=0\\x=-8[/tex]    [tex]x-5=0\\x=5[/tex]

This makes our solutions x=-8 and x=5.

If you are not familiar with factoring or the process I have went through above, I highly recommend learning about it.

Which set describes the domain of p(h)? {h| 0 ≤ h ≤ 40} {h| 0 ≤ h ≤ 60} {p(h)| 0 ≤ p(h) ≤ 1,400} {p(h)| 0 ≤ p(h) ≤ 1,800}

Answers

The set {h| 0 ≤ h ≤ 60} describe the domain of p(h).

According to the statement

We have given that the brenton gets paid $20 per hour for the first 40 hours he works in a week. For any hours above that, he is paid overtime at $30 per hour.

and we have to find that the those set which describes the domain of p(h).

So,for this purpose

In this case, the domain represents the set of hours he is permitted to work

From the question, we understand that he cannot work more than 60 hours

This means that, the least number of hours to work is 0, and the highest is 60

So, the domain is 0 to 60

When represented properly, the domain of P(h) is (b)  {h| 0 ≤ h ≤ 60}.

So, The set {h| 0 ≤ h ≤ 60} describe the domain of p(h).

Learn more about Domain here https://brainly.com/question/1770447

Disclaimer: This question was incomplete. Please find the full content below.

Question:

Brenton’s weekly pay, P(h) , in dollars, is a function of the number of hours he works, h. He gets paid $20 per hour for the first 40 hours he works in a week. For any hours above that, he is paid overtime at $30 per hour. He is not permitted to work more than 60 hours in a week. Which set describes the domain of P(h)? {h| 0 ≤ h ≤ 40} {h| 0 ≤ h ≤ 60} {P(h)| 0 ≤ P(h) ≤ 1,400} {P(h)| 0 ≤ P(h) ≤ 1,800

#SPJ4

Answer:

B.{h|0<h<60}

Step-by-step explanation:

If there are 4 green apple and 3 red apples, what's the probability that a red apple is picked?

Answers

Answer: 3/7

Step-by-step explanation:

Please help. will give brainliest

Answers

Answer:

the 4th answer option. W = 153°, P = 71°

Step-by-step explanation:

PQRS ~ TUVW

that means both figures are similar.

and that means that the angles at the corresponding vertexes are the same, and all 4 corresponding sides (and any other pair of corresponding lines like diagonals) have the same scaling factor.

we see due to the proportions of the different parts of the shapes

U corresponds to Q.

T corresponds to P.

W corresponds to S.

V corresponds to R.

therefore, as the angle at S = 153°, so is W.

and as the angle at T = 71°, so is P.

Which equation has an a-value of 1 a b-value of -3 and a c-value of -5

Answers

The equation which has the values as given in the task content is; y= x² -3x -5.

What is the quadratic equation with the given values?

By convention, the general form representation of a quadratic equations is;

y = ax² + bx +c.

Hence, it follows that when the equation has an a-value of 1 a b-value of -3 and a c-value of -5, the equation is; y= x² -3x -5.

Read more on quadratic equations;

https://brainly.com/question/1214333

#SPJ1

Other Questions
All of the following reinforce the dominance of the two major parties exceptThe increasing role played by interest groups as linkage institutionsRestrictive state ballot access lawsThe tendency of American voters to consider themselves moderateSingle-member voting districts Identical metal blocks initially at rest are released in various environments as shown in scenarios A through D below. In all cases, the blocks are released from a height of 2 m above the ground, considered to be the level of reference in this problem. If air resistance is neglected, rank the scenarios from least kinetic energy to greatest kinetic energy at the instant before the block reaches the ground. the adiabatic compressor of a refrigeration system compresses saturated r-134a vapor at 0c to 600 kpa and 50c. what is the isentropic efficiency of this compressor? An employee's current annual gross wage is $48,200.Part A: Calculate how much will be needed in retirement if the employee wants to have enough saved to live off 80% of the current annual gross wage and withdraw 4% the first year. Show all steps. Part B: The employee determines that they can contribute $400 per month to a retirement account with a 5.5% monthly compounded interest rate. Calculate the account balance if the employee plans to retire in 40 years. Show all steps.Part C: Using your values from Part A and Part B, calculate the difference between the employee's goal and the actual retirement account balance. Explain whether the employee will meet their retirement goal. a piano string of mass 10.0 g is stretched between two supports 2.0 m apart. if the tension in the string is 310 n, how long will it take a pulse to travel from one support to the other? calculate the molar solubility (mol/l) of pbcro4. Ksp = 1.8 X 10^-14 For males in a certain town, the systolic blood pressure is normally distributed with a mean of 120 and a standard deviation of 10. What is the probability that a randomly selected male's systolic blood pressure will be between 103 and 134, to the nearest thousandth? enter answer in the provided box. calculate the emf of the following concentration cell at 25c: cu(s)/cu2 (0.066 m)/ /cu2 (1.109 m)/cu(s) A copper cylinder has a mass of 76.8 g and a specific heat of 0.092 cal/gC. It is heated to 86.5 C and then put in 68.7 g of turpentine whose temperature is 19.5 C. The final temperature of the mixture is 31.9 C. What is the specific heat of the turpentine? A two lens combination consisting of a diverging Lens (#1) with a focal length of -20 cm and a converging Lens (#2) with a focal length of +30 cm is used to view the image of an object, 6 cm high, placed 30 cm in front of the first lens. The two lenses are separated by 40.0 cm. Which of the following characteristics does the final image have? how does porters model of national competitive advantage differ from the heckscher-ohlin theory? item 7 samantha would like a quick and convenient way to determine whether her distribution of body fat is unhealthy. based on this information, she should _______. Amy bought 55 lbs of clay for her art projects. She used 12.7 lbs to make a sculpture, and 0.82 lbs for each mug. How many mugs did Amy make if she had 27.54 lbs of clay left over? A streetlamp illuminates a circular area that is 23 meters across through the center. How many square meters of the street is covered by the light? Round to the nearest hundredth and approximate using = 3.14. 72.22 m2 415.27 m2 2,607.86 m2 5,215.73 m2 In 2014, a survey stated that 51% of 650 randomly sampled North Carolina residents planned to set off fireworks on July 4th. a) Determine the margin of error for the 95% confidence interval for the proportion of North Carolina residents that plan to set off fireworks. Give your answer to three decimal places. Margin of Error = _____% b) How many randomly sampled residents do we need to survey if we want the 95% margin of error to be less than 3%? Sample size > _____ People the frequency response of a system is given as vout/vin= jl / (( j)2 jr l). if l=2 h and r=1 , then what is the magnitude of the response at 70hz? G protein-Coupled Receptors (GPCRs)G proteins are all trimeric proteins made up of 3 subunits, alpha, beta and gamma.Which of these subunits are attached to the plasma membrane by a lipid linkage? Find the total of the areas under the standard normal curve to the left of z1=2.575 and to the right of z2=2.575. Round your answer to four decimal places, if necessary. Find the total of the areas under the standard normal curve to the left of z1=2.575 and to the right of z2=2.575. Round your answer to four decimal places, if necessary. the phase of the menstrual cycle that occurs between the expulsion of the ovum and the onset of menses is the: most of the products cost and functionality is determined during: a. manufacturing b. product design c. supplier selection d. launch